LSAT and Law School Admissions Forum

Get expert LSAT preparation and law school admissions advice from PowerScore Test Preparation.

 alee
  • Posts: 57
  • Joined: Mar 21, 2012
|
#4140
Hi guys,

I am pretty confused for the rationale for the correct answer to Q10, Section 4, Prep Test 54 on PAHs. The correct answer is (D):

"Most of the PAHs released into the atmosphere are the result of wear and tear on automobile tires".

How is this the 'strongest counter' to Erin's conclusion that 'the proposed regulations would save 000's of lives? Is it because it suggests that, since 'wear and tear of tyres' is inevitable. The regulations would in fact be inefficacious?

However, I am not sure how we can make these assumptions... i.e why could it not be the case that the regulations require a new type of tire design that prevents or significantly mitigates such PAH producing 'wear and tear'?

your help would be greatly appreciated, as always!
 Steve Stein
PowerScore Staff
  • PowerScore Staff
  • Posts: 1153
  • Joined: Apr 11, 2011
|
#4145
Hi,

Erin's claim is that, because ten thousand people die each year as a result of PAHs, that reducing emissions will save thousands of lives.

The right answer choice doesn't have to be a broad argument against emission limits--it just has to counter Erin's claims.

Answer choice D provides a strong counter to Erin's claims: if PAHs are mostly caused by tires--rather than emissions--then the regulation of emissions might not have the effect that Erin predicted.

Let me know whether that makes sense--thanks!

~Steve
 alee
  • Posts: 57
  • Joined: Mar 21, 2012
|
#4151
Thanks for that;

I see now that Albert refers to the 'proposed emissions regulations designed to decrease the amount of PAHs released in the atmosphere by automobile exhaust'. This means that Erin's argument is vulnerable because the regulation would only affect PAH emissions *from the exhaust*, not PAH emissions from the wear and tear of automobile tyres.

Is that what you mean?

Thanks for clarifying that for me!
 Steve Stein
PowerScore Staff
  • PowerScore Staff
  • Posts: 1153
  • Joined: Apr 11, 2011
|
#4155
That's exactly right.

~Steve
 Rita
  • Posts: 38
  • Joined: Sep 30, 2016
|
#29543
Hi Steve,

Why is answer choice E incorrect? I thought it suggested that another component in the exhaust other than PAHs may cause the cancer. If so, then implementing the regulations on PAHs could potentially save no lives at all.

Thanks,
Rita
 Claire Horan
PowerScore Staff
  • PowerScore Staff
  • Posts: 408
  • Joined: Apr 18, 2016
|
#29552
Hi Rita,

Answer choice (E) reads, "PAHs are one of several components of automobile exhaust that scientists suspect of causing cancer." If that were the case, it would not be a very effective counter to Erin's conclusion that decreasing emissions would save lives because, although other components of exhaust are also suspected, PAHs are still one of the components thought to cause cancer. Furthermore, regulating emissions would likely result in decreasing emissions from the other components, so fewer incidences of cancer would likely result from the other components as well.
 bk1111
  • Posts: 103
  • Joined: Apr 22, 2017
|
#34414
Claire Horan wrote:Hi Rita,

Answer choice (E) reads, "PAHs are one of several components of automobile exhaust that scientists suspect of causing cancer." If that were the case, it would not be a very effective counter to Erin's conclusion that decreasing emissions would save lives because, although other components of exhaust are also suspected, PAHs are still one of the components thought to cause cancer. Furthermore, regulating emissions would likely result in decreasing emissions from the other components, so fewer incidences of cancer would likely result from the other components as well.
Hello, I picked E, but upon reviewing I eliminated E because while this answer choice discusses "cancer," Erin's conclusion is limited to deaths from lung and heart diseases. Is that a proper to eliminate E?
 Kristina Moen
PowerScore Staff
  • PowerScore Staff
  • Posts: 230
  • Joined: Nov 17, 2016
|
#34418
Hi bk,

It's good to pick up on the word "cancer." Albert's argument is that there's no causal link between PAH and cancer. But Erin is talking about heart and lung disease! Further, even if Erin were talking about cancer... it doesn't matter if other components also cause cancer.
User avatar
 JoshuaDEL
  • Posts: 15
  • Joined: Apr 25, 2021
|
#89480
Hello,

I understand that D is the best counter to Erin's argument, BUT only if it was coming from someone other than Albert. Albert gives reasoning to his claim that the regulation is unnecessary by raising doubts as to whether PAHs are indeed toxic and cause cancer, mentioning that there are no causal links. If Albert said in his original argument that regulation on exhaust wouldn't provide any significant changes, then I would understand how D could be a counter.

That's why my prephrase was something like: "there are no causal links found for those diseases you mentioned too" - that's why I chose E because he at least stuck to his argument, even though I didn't like the choice myself. To me, if he were to give D as a counter in that context, it sounds like he's just switching his argument from "PAH may not be harmful" to "Oh actually, PAH is produced mostly from tire tears, not the exhaust anyways" which would make his initial argument very unconvincing.

I'm not sure if I'm focusing too much on the flow/context of the conversation but I just feel like any argument that just switches back and forth according to other's responses makes it weak. In these types of questions, do I just purely focus on the answer choice and how it weakens the respondent's argument, or do I need to focus on the connection between the answer choice and the original argument as well? In the case of the latter, what did I do wrong in this case?

Thanks so much in advance!
User avatar
 LSAT4Life
  • Posts: 17
  • Joined: Aug 10, 2021
|
#89520
Can you explain why Answer choice (C) is incorrect? I think I initially chose (C) thinking that it's saying "even if no new automobile emissions are enacted, the amount of PAH released into the atmosphere will decrease." I think this phrase by itself would've been a strong weaken response. However, the statement is later qualified by "if automobile usage declines" which is a big "if" that the stimulus does not talk about at all. I think this later phrase makes AC (C) a lot weaker as a response.

Is my thinking process correct?

Get the most out of your LSAT Prep Plus subscription.

Analyze and track your performance with our Testing and Analytics Package.